Vous êtes sur la page 1sur 51

Test #20 Q.

Id = 603 Question 1 Of 26 Division : Cardiology


Item Review
36-year-old male patient, who has a history of Marfan's syndrome, presents with
sudden onset of severe central tearing chest pain radiating to his back. The pain
is 9/10 in severity and has no relation with exertion. He is a non-smoker and non-
alcoholic. Measurement of his BP shows a difference of 35 mmHg between his
two arms. Chest auscultation shows the presence of mid-systolic click and there
is no murmur. What is the next most appropriate diagnostic step in the
management of this patient?

A. Transesophageal echocardiography
B. MRI of the chest
C. Cardiac enzymes
D. Coronary angiogram
E. Ventilation-perfusion scan
F. Arterial blood gas analysis
Explanation:

This patient is most likely suffering from acute aortic dissection for which a
transesophageal echocardiography or computed tomography of the chest are the
diagnostic studies of choice. Marfan's syndrome predisposes to the development
of aortic dissection. Tearing pain with radiation to the back and a difference in BP
of greater than 30 mmHg between two arms are important clinical clues for aortic
dissection.

MRI is used to establish the diagnosis of aortic dissection when its presentation
is chronic or when there is no hemodynamic instability.

EKG and cardiac enzymes are of great importance when coronary artery disease
is suspected. This patient has no risk factors for CAD and his pain is not typical
of CAD.

A ventilation perfusion scan is indicated when pulmonary embolism is suspected.


This patient's clinical picture is not compatible with the diagnosis of pulmonary
embolism. He has no risk factors for DVT.

ABG is not an issue here.

Educational Objective:
Transesophageal echocardiography or computed tomography are the diagnostic
studies of choice for suspected aortic dissection.

57% of people answered this question correctly

Test #20 Q.Id = 614 Question 2 Of 26 Division : Cardiology


Item Review
A 38-year-old Asian immigrant presents for the evaluation of exertional dyspnea
with minimal activity. His past medical history is significant for acute rheumatic
fever. He denies any fever, chest pain, cough, malaise or weight loss. His PR:
70/min and regular; BP: 126/76mmHg; Temperature: 37.2C(99F). His apex beat
is tapping and non-displaced. Auscultation of his lungs shows crepitations in both
lower lung fields. Auscultation of heart reveals a loud first heart sound, mid-
diastolic rumbling and a low-pitched murmur at the apex with an opening snap.
The murmur is accentuated by mild exercise. Chest X-ray shows straightening of
the left border of the heart and presence of Kerley B lines. EKG shows left atrial
enlargement. Which of the following findings is a hallmark of the suspected
disease?

A. Left atrial enlargement


B. Elevated left atrioventricular pressure gradient
C. Wide pulse pressure
D. Elevated left ventricular diastolic pressure
E. Atrial fibrillation
Explanation:

The hallmark finding of MS is elevated left atrioventricular pressure gradient that


ultimately leads to left atrial enlargement. The elevated left atrial pressures are
transmitted to the pulmonary veins, capillaries, and arteries. These phenomena
are responsible for the exertional dyspnea and later, pulmonary hypertension
and right heart failure. Enlargement of the left atrium predisposes to the
development of atrial fibrillation which is quite common in mitral stenosis but is
not the hallmark finding.

The left ventricular diastolic pressure is normal in pure mitral stenosis. These
pressures become elevated when there is coexistent mitral regurgitation, aortic
valve disease, systemic hypertension, or coronary artery disease.

A wide pulse pressure is not a feature of mitral stenosis. It occurs in aortic


insufficiency and other conditions in which circulation is hyperdynamic.

Educational Objective:
The hallmark finding of MS is elevated left atrioventricular pressure gradient.

23% of people answered this question correctly

Test #20 Q.Id = 631 Question 3 Of 26 Division : Cardiology


Item Review
A 60-year-old male presents to the emergency room with the chief complaint of
progressive dyspnea and fatigue. He denies any chest pain, syncope, cough, or
edema. He suffered an acute anterior wall MI 1 month ago. He does not have a
history of COPD or asthma. He is taking aspirin, metoprolol, enalapril, and
simvastatin. Chest auscultation reveals bilateral crackles in his lower chest.
Examination of the precordium reveals a double apical impulse. Cardiac
auscultation reveals the presence of an S3 and a pansystolic murmur at the
mitral area that radiates to the apex. Chest x-ray shows prominence of the left
heart border. EKG shows previously present unchanged Q waves and a
persistent ST segment elevation in the anterior leads. Based on these findings,
what is the most likely underlying cause of his symptoms?

A. Ventricular septal defect


B. Papillary muscle rupture
C. Free ventricular wall rupture
D. Rupture of chordae tendinae
E. Left ventricular aneurysm
Explanation:

This patient most likely has a left ventricular aneurysm, which is a late
complication of an acute MI. It usually occurs after an anterior wall MI and may
be either asymptomatic, or it may present with congestive heart failure, an
arterial embolism, or sustained ventricular arrhythmias. Precordial examination
may show a double apical beat. Auscultation of the heart may reveal an
additional S3 or S4 and sometimes a murmur of mitral regurgitation is present
due to papillary muscle dysfunction, annular dilatation, or abnormal left
ventricular geometry. Chest x-ray usually shows a characteristic prominence of
the left border of the heart. The EKG shows persistent ST segment elevation.

Ventricular septal defects and papillary muscle rupture may complicate an acute
MI. However, these complications occur much earlier within the first week post
MI and they usually present acutely with a new pansystolic murmur and
cardiogenic shock.

Rupture of chordae tendinae is not a complication of MI and it usually occurs


secondary to trauma or infective endocarditis.

Free-wall rupture is an early mechanical complication of MI. It does not produce


a murmur and it presents with cardiogenic shock.

Educational Objective:
LV aneurysm can cause CHF in a patient who sustained an anterior wall MI in
the past. A double apical beat and persistent elevation of the ST segment are
important diagnostic clues.

21% of people answered this question correctly

Test #20 Q.Id = 640 Question 4 Of 26 Division : Cardiology


Item Review
A 66-year-old white male comes for the evaluation of his progressively worsening
dyspnea and bilateral ankle swelling. He complains of dyspnea at rest,
orthopnea, and paroxysmal nocturnal dyspnea. He has a history of HTN for 15
years and suffered three MIs in the past. He was treated with a 2 vessel CABG 6
years ago and his repeat catheterization 1 year ago showed partial occlusion of
the grafts and diffuse native vessels disease. It was decided that he is not a
candidate for any intervention or surgical procedure. His ejection fraction is 30%.
His medications include aspirin, simvastatin, isosorbide, and metoprolol. On
examination, his PR: 86/min; BP: 110/70 mmHg; RR: 14/min; Temperature:
37.2C(99F). His neck veins are distended and bilateral pitting ankle edema is
present. Auscultation of his lungs is significant for the presence of bilateral
crepitations; he also has an S3 gallop without any murmurs. Chest x-ray shows
enlargement of cardiac silhouette and bilateral pulmonary edema. Addition of
which of the following medication to his regimen will increase this patient's overall
long-term survival?

A. ACE inhibitor
B. Calcium channel blockers
C. Loop diuretics
D. Digoxin
E. Dobutamine
F. Home oxygen therapy
Explanation:

Of all the above-mentioned drugs only ACE inhibitors have been shown to
decrease long-term mortality in patients with ischemic cardiomyopathy and LV
systolic dysfunction. Several randomized and meta analysis studies have proved
this benefit. ACE inhibitors should be considered in all patients with coronary
artery disease who have an EF of less than 40%.

Therapy with digoxin does not improve survival but it provides symptomatic
benefit in patients with systolic dysfunction and sinus rhythm. They are also used
to control ventricular rate in patients of heart failure who have atrial fibrillation.
Use of digoxin has been associated with less hospitalizations and lower health
care costs.

Loop diuretics are very useful in providing symptomatic relief in those patients
with heart failure who have pulmonary or peripheral edema. However, they have
not been shown to improve survival. The only diuretic, which has shown some
survival benefit in patients with severe heart failure, is spironolactone.

Calcium channel blockers have no role in the treatment of CHF and they do not
provide any survival benefit. In fact, they can worsen the peripheral edema by
causing peripheral venous dilatation.

Dobutamine is used for inpatient treatment of heart failure when it is severely


decompensated and it does not provide a survival benefit.
Home oxygen therapy has been shown to have a survival advantage in patients
who have COPD and right-sided heart failure from pulmonary hypertension.
There is no role for oxygen in left sided heart failure since the problem is not in
the lung.

Educational Objective:
Know that ACE inhibitors are the survival improving drugs in congestive heart
failure.

52% of people answered this question correctly

Test #20 Q.Id = 1704 Question 5 Of 26 Division : Cardiology


Item Review
A 67-year-old male is presented to ER because of chest pain. He has a history of
stable angina and has been on aspirin and isosorbide. His other medical
problems include HTN and bronchial asthma. Occasionally, he takes albuterol
inhalations. He is started on a nitroglycerine drip. He feels little lightheaded. His
initial EKG is shown below. His vital signs are, BP: 100/60 mm Hg, RR: 16/min
and Temperature: 37C(98F). Physical examination reveals clear chest. The next
step in his management is:

A. Urgent cardiac catheterization


B. Electrophysiology study
C. Pacemaker
D. Observation
E. Start metoprolol
Explanation:

This patient has classic EKG of third degree heart block. In third degree AV
block, no atrial impulses will travel to ventricles. So, atria and ventricles beat
independently and have their respective rates; in this case the atrial rate is
80/min and the ventricular rate is around 30/min. These patients are at very high
risk for sudden cardiac death and they should be admitted in ICU and permanent
pacemaker should be placed as soon as possible. Atropine should always be
made available at the bedside. (Option C).

Option A: Any patient who suffers an MI may undergo a cardiac catheterization


to evaluate coronary artery disease. This patient has no ‘ST’ elevation; so, in this
patient it is not urgent and can be done on an elective basis. The patient's
arrhythmia has to be addressed first as it is a more dangerous condition.

Option B: An EP study does not need to be done in all patients. This patient has
a coronary artery disease and suffered a conduction block. The EP study does
not change the treatment or the prognosis. EP studies are usually done to
identify the arrhythmia and look for the location of conduction block.

Option D: Observation alone is not indicated for any patient with complete heart
block.

Option E: Starting beta-blockers is the one thing that you can do to kill this
patient immediately; so, obviously it is not the treatment that you would want.
Beta-blockers are contraindicated in all types of heart block.

Educational objective:
Complete heart block is a dangerous condition, which can cause sudden cardiac
death. It requires immediate placement of permanent pacemaker.

52% of people answered this question correctly

Test #20 Q.Id = 976 Question 6 Of 26 Division : Cardiology


Item Review
34-year-old previously healthy male, who has recently immigrated from South
America (Brazil), presented with a 2 months history of exertional shortness of
breath. He denies any chest pain, palpitations, dizziness, or syncope. His only
other medical problem was megacolon, which was treated 2 years ago. On
examination, there are bilateral basilar crepitations heard and a 1+ pedal edema
and mild elevation of jugular venous distention. CBC revealed hemoglobin of
13.9. EKG showed nonspecific repolarization changes and intraventricular
conduction abnormalities. Chest x-ray revealed cardiomegaly and pulmonary
congestion. Echocardiogram revealed an ejection fraction of 40%. Based on
these findings, which of the following is the most likely cause of his problem?

A. Diphtheric myocarditis
B. Premature coronary artery disease
C. Trypanosoma cruzi
D. Giant cell myocarditis.
E. Rickettsial myocarditis.
Explanation:

Chaga’s disease is a condition caused by insect-borne protozoan called


Trypanosoma cruzi, which is a common form of myocarditis in Central and South
America. Major clinical manifestations may appear after a latent period of more
than a decade. At this stage, patients usually present with cardiomyopathy,
conduction abnormalities, and certain death. This patient has classic features of
cardiomyopathy. Almost all these patients will have a have a history megacolon
or mega-esophagus.

Educational Objective:
Any patient who comes from South America and have findings suggestive of
cardiomyopathy should make you think about Chaga’s disease.

70% of people answered this question correctly

Test #20 Q.Id = 1009 Question 7 Of 26 Division : Cardiology


Item Review
A 65-year-old white male who is known to have benign prostatic hyperplasia
presents to the emergency room with altered mental status. He is febrile and had
two episodes of vomiting this morning. He is also known to have type II DM for
the last seven years. His medications include aspirin, finasteride, insulin and
captopril. On examination, his vitals are as follows: BP: 70/50 mmHg,
Temperature: 40C (103F), RR 15/min, pulse rate 110/min. Significant findings on
examination are muffled heart sounds and bilateral crackles up to the scapulae.
There is no jugular venous distension or ankle edema. He is given IV antibiotics
and oxygen. An indwelling urinary catheter is passed and urine output is
measured at 20 ml/hour. Urine and blood are sent for culture. Right heart
catheterization shows pulmonary capillary wedge pressure of 30 mm Hg and a
mean right atrial pressure of 11 mmHg. Based on these findings, what is the
most likely cause of shock in this patient?

A. Right ventricular infarction


B. Pericardial tamponade
C. Constrictive pericarditis
D. Left ventricular dysfunction
E. Septic shock due to urinary tract infection
F. Hypovolemia secondary to vomiting
Explanation:

The above patient is most likely suffering from shock secondary to left ventricular
dysfunction. Shock has multiple causes and may result from hypovolemia, sepsis
and cardiac causes. The above patient’s shock may be due to any of the above-
mentioned etiologies and therefore right heart catheterization is essential to
make the diagnosis in this case.

The physical exam findings and the elevated pulmonary capillary wedge
pressure are compatible with left ventricular dysfunction.

In the case of right ventricular infarction, pulmonary capillary wedge pressure


would be low and chest examination and chest x-ray would show absence of
fluid in the lungs.

In both constrictive pericarditis and pericardial tamponade, right and left sided
diastolic pressures are equal and elevated. In the above patient, his pulmonary
capillary wedge pressure is elevated but not equal to right atrial pressure thus
excluding the diagnosis of pericardial tamponade and constrictive pericarditis.

In cases of septic shock both right atrial pressures and pulmonary capillary
wedge pressure are low.

In cases of hypovolemic shock both right atrial pressures and pulmonary


capillary wedge pressure are low.

Educational Objective:
Recognize the clinical features of cardiogenic shock. Elevated PCWP is the most
important finding to concentrate.

15% of people answered this question correctly

Test #20 Q.Id = 630 Question 8 Of 26 Division : Cardiology


Item Review
A 56-year-old white male complains of recurrent chest pain while he is in CCU
three days after a large transmural anterior wall myocardial infarction. He
received thrombolytic therapy and anticoagulation with heparin. The chest pain is
central, 6/10 in severity, radiates to the back, is worsened by deep breathing,
and relieved by sitting up. Sublingual nitroglycerin has no effect on his pain. His
vitals are, PR: 80/min; BP: 125/70 mmHg; Temperature: 37C(98.6F); RR:
14/min. His lungs are clear to auscultation. Heart sounds are normal with no
murmurs and a pericardial friction rub is present. Chest x-ray shows normal sized
heart and clear lung fields. EKG shows diffuse new ST segment elevation that is
concave upwards. Troponins are still elevated and CK-MB is normal. Based on
these findings, what is the most likely diagnosis in this patient?

A. Acute pericarditis
B. Recurrent ischemia
C. Recurrent infarction
D. Dressler’s syndrome
E. Pulmonary embolism
Explanation:

This patient is most likely suffering from acute pericarditis. Typical findings of
acute pericarditis in this patient include chest pain, worsened by breathing and
improved by leaning forward, presence of a pericardial friction rub and diffuse ST
segment elevation that is concave upwards.

Recurrent ischemia is unlikely as the cause of his chest pain. Ischemic chest
pain is not aggravated by breathing and is not relieved by sitting forward.

Recurrent infarction is unlikely as a cause for his chest pain. Pain in cases of
infarction would be of the ischemic type and CK-MB would also be elevated.
Troponins remain elevated for 1-2 weeks after an acute MI, while CK-MB returns
to normal within 48-72 hours. ST elevation will be convex upwards and is seen in
certain leads, indicating a certain territory, for example either inferior, anterior or
the lateral leads etc.

Dressler’s syndrome, an autoimmune pericarditis, is a late complication of acute


MI that usually develops between the second and tenth weeks post MI.

Pulmonary embolism is unlikely in this patient. Patients with pulmonary embolism


usually present with sudden onset of dyspnea, chest pain, tachycardia, and
tachypnea. This patient received thrombolytic therapy and anticoagulation with
heparin and is very unlikely to have a pulmonary embolism.

Educational Objective:
Recognize and differentiate acute pericarditis from other causes of recurrent
chest pain in the setting of an acute MI.

41% of people answered this question correctly

Test #20 Q.Id = 1003 Question 9 Of 26 Division : Cardiology


Item Review
A 32-year-old white female presents with dyspnea, edema and malaise. She
complains of breathlessness at rest. She also had episodes of paroxysmal
nocturnal dyspnea. She noticed bilateral ankle swelling a week ago that became
progressively worse. She is a non-smoker and non-alcoholic. Her past medical
history is not significant. She is not taking any medications. Her pulse is 90/min,
BP is 140/66 mmHg, and RR is 14/min. Examination shows jugular venous
distension, rapidly rising carotid pulse with a sudden collapse and bilateral ankle
edema. Her apical impulse is heaving and displaced inferolaterally. Auscultation
of lungs shows bilateral crepitations. Cardiac auscultation shows an S3 gallop as
well as a high-pitched, blowing, early diastolic decrescendo murmur heard best
in the left third intercostal space. Leaning forward and holding the breath in
expiration intensifies the murmur. Based on these findings, what is the most
likely diagnosis in this patient?

A. Aortic regurgitation
B. Pulmonary regurgitation
C. Mitral stenosis
D. Tricuspid stenosis
E. Aortic stenosis
F. Tricuspid regurgiation
Explanation:

The most likely diagnosis in this patient is aortic regurgitation. This patient has
features of biventricular congestive heart failure secondary to valvular disease.
Presence of collapsing (water-hammer) pulse is a clue for aortic regurgitation
and occurs due to the hyperdynamic circulation and early diastolic run-off. She
has a characteristic murmur of aortic regurgitation that is early diastolic,
decrescendo, high-pitched, blowing best heard in the left third intercostal space.
The murmur is intensified by leaning forward and holding the breath in expiration
(Valsalva).

The murmur of pulmonary regurgitation is early diastolic, decrescendo, high-


pitched, blowing, best heard along the left sternal border and it usually develops
secondary to pulmonary hypertension. It becomes more prominent with
inspiration. Also, the pulmonary component of the second heart sound is loud.

The murmur of mitral stenosis is a mid-diastolic rumble and it is best heard at the
apex.

The murmur of tricuspid stenosis is a mid-diastolic rumble and is best heard


along the left lower sternal border.

The murmur of the aortic stenosis is ejection systolic and best heard at right 2nd
intercostal space.

The murmur of tricuspid regurgiation is pansystolic.

Educational Objective:
Recognize the clinical features of aortic regurgitation.

45% of people answered this question correctly

Test #20 Q.Id = 600 Question 10 Of 26 Division : Cardiology


Item Review

A 35-year-old woman who has recently immigrated from Asia presents to the
emergency room with acute onset of dyspnea. She denies any cough, chest
pain, or fever. She has a history of rheumatic heart disease as a teen. On
examination, she has an irregular PR: 97/min; BP: 125/75 mm of Hg;
Temperature: 37.2C(98.9F). Examination of precordium shows tapping apical
impulse. First heart sound is loud and a mid-diastolic rumble is heard at the
apex. Crepitations are present in both lung fields. EKG shows an irregularly,
irregular heart rhythm and the absence of ‘P’ waves. Which of the following is
most likely the cause of abnormal heart rhythm in this patient?

A. Left atrial dilatation


B. Right atrial dilatation
C. Left ventricular dilatation
D. Left ventricular hypertrophy
E. Pulmonary hypertension
Explanation:

This patient is in congestive heart failure due to atrial fibrillation (the irregularly,
irregular heart rate and absence of ‘P’ waves) which itself is secondary to mitral
stenosis (H/O rheumatic fever, mid diastolic rumble, and loud S1). Left atrial
enlargement that results from mitral stenosis predisposes the patient to the
development of atrial fibrillation. Left atrial enlargement causes atrial fibrillation
by increasing the refractory period as well as the action potential duration and it
is responsible for all cases of atrial fibrillation occurring in the setting of MS.

Left ventricular dilatation and left ventricular hypertrophy do not occur in isolated
MS.

Right atrial dilation is a late finding in the course of MS and it occurs when right
sided heart failure takes place secondary to pulmonary hypertension. Right atrial
dilatation may cause atrial fibrillation also, but in cases of MS, the cause for atrial
fibrillation is usually left atrial dilation that occurs earlier in the course of disease.
Pulmonary HTN itself does not cause atrial fibrillation, but the right atrial
enlargement that results from it, does.

Educational Objective:
Atrial fibrillation in MS is due to left atrial dilatation.

56% of people answered this question correctly

Test #20 Q.Id = 592 Question 11 Of 26 Division : Cardiology


Item Review
A 56-year-old white male presents with dyspnea for the last 3 months. His
dyspnea was initially exertional but it has worsened progressively and now he is
breathless even at rest. He denies any chest pain or ankle swelling. He has been
smoking one-pack/day cigarettes for the last 30 years and has been drinking
alcohol heavily for the last 10 years. He is not taking any medication. His mother
died of breast cancer at 57. His vitals are, PR: 86/min, BP: 113/76mm of Hg;
Temperature: 37.1C(98.9F); RR: 13/min. On auscultation of his precordium an
S3 is heard, but there are no murmurs. Chest auscultation reveals bilateral basal
crepitations. Chest x-ray shows marked cardiac silhouette enlargement and
pulmonary venous congestion. EKG shows non-specific ST-T wave changes.
Echocardiography shows a dilated left ventricle and systolic dysfunction (EF of
25-30%). CBC shows hematocrit of 32%, WBC count of 6,000/microL, and
platelet count of 60,000/microL. Peripheral blood smear shows MCV of 101 fL.
LFTs show AST of 180U/L and ALT of 66 U/L. The findings of cardiac
catheterization and coronary angiography are not compatible with the diagnosis
of ischemic cardiomyopathy. Which of the following measures is most likely to
reverse his heart failure?

A. Cessation of cigarette smoking


B. Abstinence from alcohol
C. Reduced salt intake
D. Use of ACE inhibitors
E. Use of digoxin
Explanation:

This patient is most likely suffering from dilated cardiomyopathy secondary to


alcoholism. Findings of thrombocytopenia, macrocytosis, and elevated
transaminases are all suggestive of alcoholism in this patient. His
cardiomyopathy could be due to ischemia but coronary angiography excluded
this diagnosis in this patient. Total abstinence from alcohol is the mainstay of
alcoholic cardiomyopathy management and it may reverse this condition if it is
employed earlier in the course of the disease.

Reduced salt intake only improves symptoms of heart failure and does not
reverse the disease process itself.

Digitalis is most useful in those heart failure patients who have systolic
dysfunction and have rapid ventricular rates due to atrial flutter or atrial
fibrillation. It proves to be useful in these cases by its positive inotropic effect and
negative dromotropic (slowing AV conduction). Use of digitalis has not been
shown to provide any survival advantage in patients with congestive heart failure
and has not been shown to reverse the disease process.

ACE inhibitors have been shown to slow the progression of heart failure but they
don’t reverse the disease process of alcoholic cardiomyopathy. ACE inhibitors
should be given to all patients with heart failure who have systolic dysfunction
unless they are contraindicated or the patients can’t tolerate them.

Cigarette smoking is a risk factor for coronary heart disease and its cessation
should be encouraged in all patients with heart failure. Cessation of cigarette
smoking does not reverse the disease process of alcoholic cardiomyopathy.

Educational Objective:
Know how to diagnose and manage a patient with congestive heart failure due to
alcoholic dilated cardiomyopathy.
34% of people answered this question correctly

Test #20 Q.Id = 606 Question 12 Of 26 Division : Cardiology


Item Review
A 40-year old female immigrant from Asia comes for evaluation of her dyspnea.
Her dyspnea was mild 6 months ago but it has now progressed to dyspnea, even
at rest. She denies any chest pain, syncope, or palpitations. She is a non-smoker
and non-alcoholic. Her past medical history is significant for pulmonary
tuberculosis. Her PR: 82/min; BP: 135/70 mmHg; Temperature: 37.1C(98.8F);
RR: 14/min. On her examination, significant physical findings are jugular venous
distension, bilateral ankle edema, and tender hepatomegaly. Chest x-ray shows
pericardial calcifications. Which of the following set of physical findings is most
likely to be present in this patient?

A. Early third heart sound and inspiratory increase in jugular venous pulse
B. Water hammer pulse and pistol shot femorals
C. Tapping apex beat and malar flush
D. Pulsus paradoxus and hypotension
E. Pansystolic murmur at left lower sternal border
Explanation:

This patient, most likely, has constrictive pericarditis. The finding of pericardial
calcifications on the chest X-ray is an important clue. The etiology of pericarditis
is probably from her prior history of Tuberculosis. The early third heart sound,
that is also called pericardial knock and the inspiratory increase in the jugular
venous pressure (Kussmaul’s sign), are important physical findings of
constrictive pericarditis. Kussmaul’s sign is also present in right sided heart
failure, severe tricuspid regurgitation, right ventricular infarction and cardiac
tamponade.

Water hammer or collapsing pulse and pistol shot femoral pulses are diagnostic
clues to aortic regurgitation. These physical findings occur due to a
hyperdynamic circulation and early diastolic runoff of aortic insufficiency.

Tapping apex beat and malar flush are important physical findings of mitral
stenosis. Pulsus paradoxus is defined as greater than 10-mmHg fall of the
systolic blood pressure during inspiration.

Pulsus paradoxus and hypotension point toward the diagnosis of pericardial


tamponade. The former is uncommon in constrictive pericarditis unless an
effusion is present. It may also be present in severe airway obstruction and
superior vena cava obstruction.

A pansystolic murmur at the left sternal border is usually seen in tricuspid


regurgitation.
Educational Objective:
Know the characteristic clinical findings of constrictive pericarditis.

35% of people answered this question correctly

Test #20 Q.Id = 1746 Question 13 Of 26 Division : Cardiology


Item Review
patient is referred to the urologist for urethral stricture dilatation which may result
in bacteremia and cause infective endocarditis in predisposed patients. Which of
the following patients would not require antibiotic prophylaxis to prevent infective
endocarditis?

A. A 50-year-old male with pacemaker


B. A 17-year-old male with cyanotic heart disease
C. A 50-year- old female with previous infective endocarditis
D. A 40-year-old male with mitral valve prolapse with regurgitation
E. A 30-year-old male with prosthetic aortic valve
Explanation:

The need of antibiotic prophylaxis for IE depends on the cardiac abnormality and
the procedure causing bacteremia. Certain cardiac conditions are more
susceptible to endocarditis than others. The risk of bacteremia and the likely
organisms vary according to the procedure being performed. Therefore, the
decision whether or not to prophylax, and the choice of antibiotics, depends both
on the cardiac abnormality and on the procedure.

Cardiac abnormalities are high risk, moderate risk or negligible risk. Antibiotic
regimen is different for GI and GU procedures in high-risk and moderate-risk
patients. Antibiotic prophylaxis is not recommended in moderate-risk patients
with some low-risk procedures. Generally, we can say that prophylaxis is
recommended for high-risk conditions and is optional for moderate-risk
conditions.

Conditions, which put a patient at high risk to develop infective endocarditis


include:

1. All prosthetic heart valves.


2. Any history of previous bacterial endocarditis.
3. Complex cyanotic congenital heart disease and surgically- constructed
systemic pulmonary shunts.

Conditions, which put a patient at moderate risk of developing infective


endocarditis include:
1. Congenital cardiac malformations not falling into the high or negligible risk
categories (such as PDA, VSD, ostium primum ASD, bicuspid aortic valve
and coarctation).
2. Acquired valvular heart disease (such as rheumatic heart disease,
valvular stenosis and regurgitation).
3. MVP with regurgitation and/or myxomatous leaflets.
4. Hypertrophic cardiomyopathy.

Conditions that do not require IE prophylaxis include:

1. Isolated ostium secundum ASD and surgically-repaired ASD, VSD and


PDA (beyond six months and without sequelae).
2. Mitral valve prolapse without mitral regurgitation and without thickened
leaflets.
3. Innocent or physiologic murmurs (echo required in the adult population to
rule out valvular lesion).
4. Cardiac pacemakers and defibrillators.
5. History of isolated bypass surgery, history of Kawasaki disease without
valvular dysfunction and history of rheumatic fever without valvular
dysfunction.

Educational Objective:
Patients with artificial pacemakers and defibrillators do not require prophylaxis for
infective endocarditis.

35% of people answered this question correctly

Test #20 Q.Id = 634 Question 14 Of 26 Division : Cardiology


Item Review
A 76-year-old male comes to the emergency room with sudden onset of severe
chest pain. The pain is 10/10 in severity, tearing in quality, and radiates to the
back. He is a known diabetic for 10 years and is hypertensive for 8 years. His
medications include insulin, metoprolol and hydrochlorothiazide. His vitals are,
PR: 86/min; BP: 166/96 mmHg in his right arm and 136/70 in his left arm;
Temperature: 37C(98.6F); RR: 14/min. On auscultation, lung fields are clear and
heart sounds are normal without any murmurs. Chest x-ray and EKG are normal.
Bases on these findings, which of the following is the most likely diagnosis of this
patient?

A. Angina pectoris
B. Myocardial infarction
C. Acute pericarditis
D. Aortic dissection
E. Pulmonary embolism
Explanation:

This patient is most likely suffering from aortic dissection. The most frequent
predisposing condition for aortic dissection is hypertension. Sudden onset of
severe tearing pain, with radiation to the back, is a typical presentation of aortic
dissection. The finding of a difference of more than 30 mmHg in the blood
pressure readings between two arms is another important clue for the diagnosis
of aortic dissection.

Pain of IHD usually feels like pressure and it typically radiates to the jaw, left
shoulder or left arm. Pain of angina occurs with exertion and is relieved with rest
and sublingual nitroglycerin. Pain of MI occurs at rest and is not fully relieved by
sublingual nitroglycerin. EKG usually provides evidence for ischemic heart
disease with T-wave inversion in angina pectoris, and ST segment elevation or
ST segment depression in cases of MI. The nature of pain in the above patient
with absent EKG changes makes the diagnosis of angina, or MI, very unlikely.

Pain in acute pericarditis is pleuritic, is worsened by deep breathing and


coughing and is alleviated by leaning forward. EKG may show ST segment
elevation that is concave upwards without any reciprocal changes.

Pulmonary embolism usually presents with sudden onset of dyspnea, chest pain,
tachycardia, and tachypnea. There is usually a history of risk factors like
immobilization, surgery of hip or femur etc. that predisposes the development of
pulmonary embolism.

Educational Objective:
Suspect aortic dissection as a cause of tearing chest pain in the setting of HTN
and BP difference in the 2 arms.

69% of people answered this question correctly

Test #20 Q.Id = 1634 Question 15 Of 26 Division : Cardiology


Item Review
A 72-year-old female is admitted to the ICU with severe chest pain. The initial set
of cardiac enzymes are positive and her ECG reveals an anterior wall MI. She is
being treated with aspirin, t-PA, metoprolol, nitroglycerine drip, and morphine.
Two hours later, her telemetry monitor rhythm changes and is shown below. The
next step in her management is:

A. IV lidocaine
B. Defibrillation
C. IV digoxin
D. IV amiodarone
E. Stat echocardiogram
F. STAT check of potassium and magnesium
Explanation:

Ventricular fibrillation can occur as a consequence of or in association with a


serious illness, cardiovascular disease or non-cardiovascular disorder.
Ventricular fibrillation reflects a total disorganization of ventricular electrical
activity and is recognized on the ECG by fibrillatory waves and absence of
regular QRS waves. It should be differentiated from cardiac asystole, which is
characterized by a flat base line. All patients have no BP at the onset of
ventricular fibrillation and the most immediate step is to defibrillate as soon as
possible. The energy required may be 200-360 joules. More than a few minutes
of delay can lead to permanent brain injury and death (Option B).

(Option A): Lidocaine is an excellent agent for ventricular arrhythmias. In the


presence of ventricular fibrillation, cardioversion is the first modality of therapy. If
cardioversion does not initially work, patients may require a lidocaine bolus
followed by a second shock.

(Option C): Digoxin has no role in ventricular fibrillation.

(Option D): Amiodarone is an excellent drug for stable ventricular tachycardia


and maintaining normal sinus rhythm following a ventricular arrhythmia.
However, when ventricular fibrillation occurs, the first choice is cardioversion.
Amiodarone can be loaded to maintain the normal sinus rhythm. Amiodarone is
being used more frequently (drug of choice) over lidocaine for ventricular
arrhythmias.

(Option E): Echo may be done later to assess the heart function but it does not
change the management of ventricular fibrillation. Ventricular arrhythmias are
more prone in patients with ischemic heart disease and in those with low ejection
fractions.

(Option F): When ventricular fibrillation occurs, the first thing is to defibrillate the
patient. Electrolytes and blood gas analysis can be done during the procedure
but should not delay shocking. Ventricular arrhythmias may occur in the
presence of hypokalemia.

Educational objective:
The treatment of ventricular fibrillation is STAT defibrillation with 200-360 joules.
If defibrillation fails, lidocaine or amiodarone (drug of choice) can be loaded and
the patient shocked again. Epinephrine can sensitize the heart and lower the
threshold for conversion.
35% of people answered this question correctly

Test #20 Q.Id = 1856 Question 16 Of 26 Division : Cardiology


Item Review
50-year-old woman presents to your office complaining of lower extremity edema
that started several weeks ago, and slowly progressed thereafter. Her past
medical history is significant for hypertension, treated with metoprolol for 2
years. Amlodipine was added recently because of inadequate control of BP with
metoprolol alone. She does not smoke or consume alcohol. She has no known
drug allergies. Her blood pressure is 130/80mm Hg and her heart rate is 64/min.
The physical examination reveal bilateral symmetric 3+ pitting edema of both
lower extremities, without any skin changes or varicosities. Her neck vein
pulsation is normal. Other physical findings are within normal limits. Her
laboratory studies reveal the following:

Serum albumin 4.5 g/dL


Total serum bilirubin 0.8 mg/dL
Serum sodium 140 mEq/L
Serum potassium 4.0 mEq/L
Serum creatinine 0.8 mg/dL

Urinalysis is within normal limits. What is the most likely cause of the edema in
this patient?

A. Heart failure
B. Liver disease
C. Renal disease
D. Venous insufficiency
E. Side effect of her medications
Explanation:

It is important to remember that peripheral edema is a common side effect of the


treatment with dihydropyridine Ca-channel antagonists like amlodipine. These
medications can cause significant edema, as in this patient, due to the property
to dilate peripheral blood vessels; it is NOT an allergic reaction! If the edema is
significant, the drug should be discontinued. The clues to the correct diagnosis in
this case are the therapy with a dihydropyridine Ca-channel antagonist,
amlodipine initiated recently, and normal physical examination with
normal laboratory findings. Of course, it is necessary to exclude other more
serious causes of edema.

Congestive heart failure (Choice A) is an important cause of lower extremity


edema, but the absence of other symptoms of heart failure (e.g., dyspnea,
orthopnea) and physical findings, especially elevated neck vein pulsation and
liver enlargement, makes this diagnosis unlikely.

Liver diseases (Choice B) can cause lower extremity edema, but usually ascites
dominates over peripheral edema, and abnormal laboratory findings
characteristic of liver dysfunction are present (e.g., hypoalbuminemia,
hyperbilirubinemia).

Renal diseases (Choice C) can cause peripheral edema due to massive


proteinuria (as with nephrotic syndrome) or fluid retention (such as during acute
nephritic syndrome). The absence of proteinuria, hypoalbuminemia, and normal
creatinine level are inconsistent with renal causes of the edema.

Venous insufficiency (Choice D) is a possibility in this case. Symmetric


edema without skin changes and varicosities, however, argues against venous
insufficiency.

Educational objective:
Dihydropyridine Ca-channel antagonists can cause peripheral edema and
should always be considered in the differential diagnosis of this condition, along
with other causes, such as heart failure, renal disease and venous insufficiency.

36% of people answered this question correctly

Test #20 Q.Id = 612 Question 17 Of 26 Division : Cardiology


Item Review
A 15-year-old white male, who has recently immigrated from Asia, is found to
have a heart murmur during his routine physical examination. He has a
documented history of acute rheumatic fever, without any evidence of carditis, a
year ago. He is otherwise completely asymptomatic. His vitals are stable. His
apex beat is tapping. Cardiac auscultation shows a loud first heart sound,
opening snap and mid-diastolic rumble at the apex. His lungs are clear to
auscultation. Chest x-ray and EKG are unremarkable. What is the next most
appropriate step in the management of this patient?

A. Penicillin prophylaxis
B. Refer to cardiology for balloon valvotomy
C. Refer to cardiothoracic surgeon for mitral valve replacement
D. Consider anticoagulation
Explanation:

This patient is suffering from rheumatic heart disease with pure mitral stenosis.
Although he is asymptomatic at this stage, his disease is likely to be progressive.
It has been found that prevention of recurrent attacks of rheumatic fever may
slow down the progression of mitral stenosis. Therefore, penicillin prophylaxis
with monthly IM injection of benzathine penicillin is recommended in adolescent
patients in whom the recurrence risk is high. Patients with, or without, carditis
need penicillin prophylaxis, although the recommended duration of prophylaxis is
10 years in the former and 5 years in the latter. Patients with mitral stenosis are
also at high risk for thromboembolism. Anticoagulation is indicated if they have a
history of an embolic event or they have intermittent or chronic atrial fibrillation.

Surgical intervention for mitral stenosis is indicated when these patients develop
symptoms of NYHA functional class III or IV. Balloon valvotomy is the procedure
of choice when there is favorable valve morphology.

Mitral valve replacement is considered to be the last resort for the treatment of
MS because of the high perioperative mortality and morbidity.

Educational Objective:
Know that prevention of recurrent attacks of rheumatic fever with antibiotic
prophylaxis may slow down the progression of mitral stenosis in adolescents.
Asymptomatic MS do not require any treatment except penicillin prophylaxis.

46% of people answered this question correctly

Test #20 Q.Id = 17 Question 18 Of 26 Division : Cardiology


Item Review
A 26-year-old primigravida at 20th week gestation presents to the emergency
room with a sudden onset of tearing chest pain radiating to her back and left arm.
The patient is pale and diaphoretic. Her PR: 116/min; BP: 190/100 in left arm,
and 80/60 in right arm; RR: 36/min. Her cardiac examination reveals a diastolic
murmur along the left sternal border. Her previous prenatal care is not known.
She is a smoker with a 10 pack/year history and drinks alcohol. Her ECG reveals
mild left axis deviation and ST segment depression in lead II, III, and AVF. What
is the most appropriate next step in the management of this patient?

A. Obtain CK-MB and Troponin levels


B. Transesophageal echocardiogram (TEE)
C. Transthoracic echocardiogram (TTE)
D. Antihypertensive treatment
E. CT scan of chest
F. Chest - X ray
Explanation:

Patient is most likely suffering from dissection of the aorta. Pregnancy is


associated with dissection of aorta, probably due to changes in connective tissue
associated with pregnancy. Other risk factors for dissection include a bicuspid
aortic valve, coarctation of aorta, and Marfan’s syndrome. Aortic dissection is a
medical emergency and should be diagnosed and treated. If hypertension is
present it should be treated aggressively before any diagnostic studies are done.
ECG changes are due to involvement of the coronary ostia (usually the right
coronary artery), which lead to inferior wall ischemia, manifested by the
abnormalities on ECG.

Choice (A): Obtaining cardiac enzymes is incorrect because the patient doesn’t
have any risk factors for an MI. Her ECG changes are secondary to ischemia
and not due to infarction.

Choice (B): TEE is the initial investigation of choice to diagnose suspected aortic
dissection. But it is indicated only after hypertension treatment is initiated.

Choice (C): Transthoracic echocardiogram is not used to diagnose aortic


dissection.

Choice (E): CT scan of chest also has high sensitivity and specificity and can be
used instead of the transesophageal echo.

Choice (F): Chest - X ray also reveals aortic dissection as mediastinal


enlargement but still not the 1st step in this patient.

Educational Objective:
Antihypertensive management should be the first step in patients with aortic
dissection with hypertension.

24% of people answered this question correctly

Test #20 Q.Id = 589 Question 19 Of 26 Division : Cardiology


Item Review
A 20-year-old white male comes with the complaint of exertional dyspnea for the
last 2 months. He denies any chest pain, cough, fever or syncope. He is not
taking any medicines. He does not smoke and drinks alcohol only on weekends.
One of his brothers died of a heart disease when he was 22. His vitals are stable
and he is afebrile. On examination, there is no edema, jugular venous distension
or hepatomegaly. His lungs are clear to auscultation. On auscultation of the
precordium, a harsh crescendo-decrescendo grade III systolic murmur is heard
most prominently along the left lower sternal border. The murmur is intensified by
Valsalva maneuver and attenuated by leg elevation. What should you order next
in this patient?

A. Electrocardiogram
B. Echocardiography
C. Cardiac catheterization
D. Stress testing
E. MRI
F. Exercise thallium SPECT imaging
Explanation:

This patient is most likely suffering from hypertrophic cardiomyopathy (HCM).


HCM can be asymptomatic or it may present with symptoms of exertional
dyspnea, chest pain or syncope. The typical auscultatory finding is a harsh
diamond shaped systolic murmur best heard at the left lower sternal border. The
murmur of HCM increases in intensity with the Valsalva maneuver and is
attenuated by leg elevation. The best way to diagnose HCM is
echocardiography, which usually shows asymmetrical interventricular septal
hypertrophy. In most cases, the thickness of the septum is 1.3 times greater the
thickness of left ventricular free wall. Systolic anterior motion of the mitral valve is
noted if an outflow pressure gradient is present.

Cardiac catheterization is not required for the diagnosis of HCM but it helps to
quantify the pressure gradient and to exclude coexistent coronary artery disease
as a cause of chest pain in older patients.

ECG shows evidence of left ventricular hypertrophy but it is insufficient to make a


diagnosis of HCM. Other abnormal ECG findings include prominent Q waves in
inferior and lateral leads due to septal depolarization of the hypertrophied
myocardium.

Exercise testing is not diagnostic of HCM but it provides a measure of patient’s


physical limitations.

In some cases, echocardiography fails to establish wall thickness and extent of


hypertrophy. MRI is an alternative diagnostic study for HCM when
echocardiography is suboptimal.

Exercise thallium scintigraphy is useful for detecting ischemic but viable


myocardium.

Educational Objective:
The investigation of choice for the diagnosis of HCM is echocardiography.

66% of people answered this question correctly

Test #20 Q.Id = 632 Question 20 Of 26 Division : Cardiology


Item Review
A 63-year-old white male presents to the ER with sudden onset of severe
retrosternal chest pain associated with nausea, vomiting and diaphoresis. His
EKG shows ST segment elevation in leads aVL and I. The patient was treated
with thrombolytic therapy in the ED. He is known to have insulin requiring
diabetes mellitus for the past 3 years as well as hypertension for the past 2
years. His medications include insulin and enalapril. Which of the following set of
medications has been shown to reduce mortality after an acute MI?

A. Nitrates, beta blockers, and aspirin


B. ACE inhibitors, nitrates, and beta blockers
C. Magnesium, beta blockers, and aspirin
D. Oxygen, morphine, aspirin, and nitrates
E. Aspirin, beta blockers, and ACE inhibitors
Explanation:

Of all the above choices, only ACE inhibitors, aspirin, and beta-blockers have
been shown to reduce mortality after acute MI.

The proposed mechanism for ACE inhibitors is the reduction of ventricular


remodeling and thus reduced incidence of left ventricular dysfunction /
aneurysmal dilatation after MI. They are recommended in all patients of acute MI
with CHF as well as to those who are hemodynamically stable and have ST
segment elevation or left bundle branch block. They are usually started within the
first 24 hours of acute MI and are continued indefinitely in those who have CHF,
HTN or LV dysfunction.

Aspirin has also been shown to be effective and should be given in all patients,
as early as possible in the setting of an acute MI, because it has been shown to
reduce mortality by as much as 30%.

3 doses of IV beta-blockers, followed by oral beta-blockers in doses titrated for a


heart rate of 50-60, have also been shown to reduce the mortality significantly in
acute coronary syndromes.

Morphine is an effective analgesic and is used for pain relief in the setting of an
acute MI. Oxygen is useful only when pulse oximetry shows hypoxemia. Its
routine use in all patients of acute MI is not cost effective. Nitrates are used only
for pain relief in the setting of acute MI and they don’t incur any survival
advantage. IV nitrates should be avoided when there is hypotension or evidence
of right ventricular infarction. IV magnesium is indicated only when initial testing
shows low serum magnesium level. Routine use of IV magnesium in all MI
patients does not provide any survival benefit.

Educational Objective:
Aspirin, ACE inhibitors, and beta-blockers have been shown to reduce mortality
in the setting of acute MI.

51% of people answered this question correctly

Test #20 Q.Id = 594 Question 21 Of 26 Division : Cardiology


Item Review
A 60-year-old female presents with dyspnea and ankle edema. Her dyspnea was
mild 6 months ago and used to occur with moderate exertion but now has
progressed to dyspnea, even at rest. She denies any chest pain, syncope, or
palpitations. She does not smoke or drink alcohol. She denies diabetes mellitus,
hypertension, or hyperlipidemia. Her vitals are, PR: 80/min; BP: 130/70 mmHg;
Temperature: 37.1C(98.9); RR: 14/min. Examination is significant for jugular
venous distension, bilateral ankle edema, and tender hepatomegaly. Kussmaul’s
sign is positive and pulsus paradoxus is negative. Chest auscultation shows
bibasilar crackles. Heart sounds are distant and there is no murmur. Chest x-ray
shows mild cardiomegaly and a right-sided pleural effusion. EKG shows low
voltage QRS complexes and nonspecific ST-T wave changes. Echocardiography
shows symmetrical thickening of left ventricular walls, normal ventricular size,
and slightly reduced systolic function. CT scan of the chest shows normal
pericardial thickness. Based on these findings, which of the following is the most
likely diagnosis in this patient?

A. Dilated cardiomyopathy
B. Restrictive cardiomyopathy
C. Hypertrophic cardiomyopathy
D. Cardiac tamponade
E. Constrictive pericarditis
Explanation:

Restrictive cardiomyopathy is characterized by severe diastolic dysfunction due


to a stiff ventricular wall. Chest x-ray shows only mild enlargement of the cardiac
silhouette. Echocardiography usually shows a symmetrically thickened ventricle
wall, normal or slightly reduced left ventricle size and normal or near normal
systolic function. Kussmaul’s sign may also be present. The apical impulse is
easily palpable in restrictive cardiomyopathy as opposed to constrictive
pericarditis. Restrictive cardiomyopathy is difficult to differentiate from
constrictive pericarditis. With constrictive pericarditis, chest x-rays may show
pericardial calcifications and the CT scan usually shows increased thickness of
pericardium. Kussmaul’s sign may be positive in both conditions. With
constrictive pericarditis, the thickness of myocardium will be normal.

Dilated cardiomyopathy is characterized by impaired systolic function of left and


right ventricle leading to progressive cardiac enlargement. Chest x-ray shows
marked or moderate enlargement of cardiac silhouette. Echocardiography shows
systolic dysfunction and left ventricular dilatation with normal thickness of the
ventricular wall.

Hypertrophic cardiomyopathy is characterized by asymmetric left ventricular


hypertrophy. In HCM, a harsh systolic murmur best heard at the left sternal
border is also present. Chest x-ray shows mild enlargement of cardiac silhouette.
Echocardiography shows vigorous systolic function, asymmetric septal
hypertrophy and in some cases systolic anterior motion of the mitral valve. Due
to the hypertrophy of the left ventricular wall, there is diastolic dysfunction.

In cardiac tamponade, there is sinus tachycardia and hypotension. Pulsus


paradoxus is also present in this condition. There is jugular venous distension
with a prominent 'y' descent.

Educational Objective:
Know how to diagnose restrictive cardiomyopathy and how to differentiate it from
other cardiac conditions that may present similarly.

38% of people answered this question correctly

Test #20 Q.Id = 1713 Question 22 Of 26 Division : Cardiology


Item Review
A 55-year-old male is admitted to the ICU after being involved in a motor vehicle
accident. He required an exploratory laparotomy for a bowel perforation. Two
days after surgery he remains hypotensive and requires both volume and
pressor support. At night, the nurse informs you that the patient’s fingers are blue
and cold. The photograph is shown below. Which of the following is the most
liklely cause of this finding?

A. Septic embolism
B. Raynaud’s syndrome
C. Norepinephrine
D. Aortic occlusion
E. Thrombosis of forearm vessels
Explanation:

The above presents a case of a patient who is hypotensive and receives fluid
and pressor support. Later, he is found to have all digits blue. The most likely
cause of this patient’s pathology is related to use of norepinephrine.
Norepinephrine is a powerful vasoconstrictor and can lead to decreased blood
supply to both the lower and upper extremity. Norepinephrine is a powerful
pressor drug and frequently used to revive patients in shock. In addition to
decreasing blood supply to the digits, other organs like the kidneys can also be
affected. For this reason, the use of this vasoconstrictor is limited in the ICU.
Patients who have prior atherosclerotic and peripheral vascular disease are most
sensitive to norepinephrine (Option C).

Option A: Emboli to all five digits of the hand is very rare. Embolus in the upper
extremity is very rare and is occasionally related to procedures done in the
brachial artery. Emboli usually affects one or two digits and only the tips of the
finger will be blue. Large emboli will usually affect the entire blood supply to the
radial or ulnar arteries and the bluishness/cyanosis will be visible at the wrist or
forearm. Even a complete occlusion of the radial artery is well tolerated by most
patients.

Option B: Raynaud’s syndrome is seen in a few vasculitic conditions and is due


to spasm of the vessels. The condition is not selective to one hand only and is
worse in cold weather. Most patients have a history of lupus, scleroderma or
CREST. Raynaud’s is typically seen in females. Diagnosis is based on clinical
findings and treatment is the use of calcium channel blockers.

Option D: Occlusion of the aorta does not produce selective bluishness of the
fingers. Sometimes, occlusion of the subclavian artery can occur in thoracic
outlet syndrome and leads to a cool hand. In this case, the entire hand is cold
and painful, requiring urgent treatment. Occlusion of the aorta generally occurs in
the abdominal area and leads to decreased blood flow to the lower extremities.
Acute aortic occlusion can compromise blood flow to the lower extremities and
abdominal organs.

Option E: Thrombosis of the forearm vessels is very rare and does not present
acutely with a cold hand. Thrombosis of the forearm vessels may be seen in
Buerger’s disease. Buerger’s disease is seen in young males who are heavy
smokers. Digital ischemia and gangrene are common features.

Educational Objective:
Bluish discoloration and cool fingers in the ICU are a common finding after use of
norepinephrine for hypotension.

12% of people answered this question correctly

Test #20 Q.Id = 1008 Question 23 Of 26 Division : Cardiology


Item Review
A 64-year-old white female who is known to have congestive heart failure
presents for the evaluation of nausea, vomiting, vague abdominal pain, diarrhea,
fatigue and blurred vision for the last one-day. She denies any worsening of her
breathlessness, chest pain, fever, productive cough or urinary complaints. She
had an anterior MI one year ago and later on developed congestive cardiac
failure secondary to left ventricular systolic dysfunction. She is currently on
aspirin, digoxin, furosemide, enalapril and metoprolol. On examination her vitals
are B.P: 120/80 mm Hg, R.R: 15/min, PR: 66/min. Physical examination is
unremarkable. Chest x-ray shows enlargement of cardiac silhouette and normal
lung fields. She is suspected to have digitalis toxicity and her blood is sent for
digoxin levels. EKG is ordered. Which of the following EKG finding is most
specific for digitalis toxicity?
A. Atrial flutter
B. Atrial fibrillation
C. Mobitz type II AV block
D. ST segment depression
E. First degree AV block
F. Atrial tachycardia with AV block
Explanation:

Digitalis produces a number of EKG abnormalities and arrhythmias. Some of


these abnormalities occur at therapeutic serum levels of digoxin and are called
digitalis effect. Other EKG abnormalities occur at toxic serum levels of digoxin.
Digitalis is a frequently used drug in the treatment of congestive cardiac failure
and signs of its toxicity including EKG findings are very important. Digitalis
produces these EKG abnormalities and arrhythmias due to its depressant action
on AV node and SA node and by increasing myocardial automaticity.
ST segment depression, T wave inversion and first degree AV block can occur at
therapeutic levels of digoxin and they do not represent digitalis toxicity and
therefore there is no need for discontinuation of the drug.

Atrial flutter, atrial fibrillation and Mobitz type II block are rare arrhythmias
induced by digitalis toxicity.

Atrial tachycardia along with variable degree of AV block is the most important
EKG finding of digitalis toxicity and it has a high specificity for digitalis toxicity.

Educational Objective
Know the electrocardiographic manifestations of digitalis toxicity.

28% of people answered this question correctly

Test #20 Q.Id = 591 Question 24 Of 26 Division : Cardiology


Item Review
40-year-old African American male presents with two episodes of syncope after
strenuous exercise. He also complains of breathlessness on exertion. He quit
smoking 15 years ago and he does not drink alcohol. He is not taking any
medication. He denies any chest pain, cough, or fever. His vitals are stable and
he is afebrile. On examination there is no edema, ascites, hepatomegaly or
jugular venous distension. His lungs are clear to auscultation. Auscultation of his
precordium shows a harsh diamond shaped systolic murmur best heard at the
left lower sternal border. A diagnosis of hypertrophic cardiomyopathy is
established based on echocardiogram findings. Which of the following is true
about the murmur of hypertrophic cardiomyopathy?

A. Decrease in intensity by Valsalva


B. Decrease in intensity by handgrip
C. Decrease in intensity by standing after squatting
D. Increase in intensity by phenylephrine
E. Increase in intensity by leg elevation
Explanation:

In almost 25% of cases of hypertrophic cardiomyopathy there is dynamic


obstruction to left ventricular outflow tract caused by apposition of the anterior
mitral leaflet to the septum (the systolic anterior motion of the mitral valve on the
echocardiogram). As a result of this dynamic obstruction, filling pressures are
further elevated and forward output is compromised. This outflow gradient is
increased by maneuvers that reduce the cavity size of left ventricle. Valsalva
maneuver and standing after squatting are two maneuvers that decrease left
ventricle volume thus increasing the gradient and intensifying the associated
systolic murmur. On the other hand, handgrip increases the systemic arterial
resistance and thus decreases the gradient and associated systolic murmur.
Phenylephrine also decreases the murmur by increasing systemic arterial
pressure. Leg elevation increases the left ventricular volume and thus decreases
the gradient and the associated murmur.

Educational Objective:
Know the physiology of HCM and be able to correctly identify the factors that
increase or decrease the intensity of the murmur in HCM.

38% of people answered this question correctly

Test #20 Q.Id = 1997 Question 25 Of 26 Division : Cardiology


Item Review
32-year-old Caucasian female comes to the physician because of a one-week
history of fatigue, progressive worsening of shortness of breath, and swelling of
feet. She denies any chest pain. She has no other medical problems except a
recent cold two weeks ago. She is not taking any medications. Her temperature
is 36.7 C (98 F), blood pressure is 110/65 mmHg, pulse is 90/min, and
respirations are 20/min. Bilateral basal crackles, elevated jugular venous
pressure, and 2+ bilateral pitting edema of the ankles are noted. Complete blood
count is unremarkable. Transthoracic echocardiogram of her heart will most likely
show?

A. Concentric hypertrophy of the heart


B. Eccentric hypertrophy of the heart
C. Mitral stenosis
D. Hypokinesia of the inferior wall
E. Vegetations on aortic valve
F. Asymmetric septal hypertrophy
G. Dilated ventricles with diffuse hypokinesia
H. Primary pulmonary hypertension
Explanation:

The clinical presentation of this patient is suggestive of congestive heart failure


(CHF). A history of recent upper respiratory tract infection followed by sudden
onset of cardiac failure in an otherwise healthy patient is suggestive of dilated
cardiomyopathy, most likely secondary to acute viral myocarditis.

Dilated cardiomyopathy is the end result of myocardial damage produced by a


variety of toxic, metabolic, or infectious agents. Viral or idiopathic myocarditis is
most commonly seen following Coxsackie B infection. Viral myocarditis is seen in
about 3.5-5 % of patients infected with Coxsackie B virus. Other viruses
implicated are adenovirus, cytomegalovirus, echovirus, hepatitis C, influenza
virus, parvovirus B-19, and Epstein-Barr virus. The diagnosis is made by
echocardiogram, which typically shows dilated ventricles with diffuse hypokinesia
resulting in low ejection fraction (systolic dysfunction). Viral myocarditis can
cause dilated cardiomyopathy by direct viral damage, as well as sequel of
humoral or cellular immune responses to persistent viral infection.

(Choice A) Concentric hypertrophy of the heart is seen following chronic


pressure overload, as in valvular aortic stenosis or untreated hypertension;
however, these are chronic conditions and would not have acute onset of heart
failure.

(Choice B) Eccentric hypertrophy of the heart is seen following chronic volume


overload, as seen in valvular regurgitation; however, this is a chronic condition
and would not have acute onset of heart failure.

(Choice C) Mitral stenosis is characterized by middiastolic murmur and opening


snap. EKG shows left atrial hypertrophy. Chronic mitral stenosis would cause
right ventricular hypertrophy and, thus, right ventricular heave secondary to
pulmonary hypertension.

(Choice D) Hypokinesia of the inferior wall would be seen in case of inferior wall
myocardial infarction, but clinical presentation of this patient is not suggestive of
myocardial infarction.

(Choice E) Aortic valve vegetation can cause aortic regurgitation, which, if acute,
would not have dilated heart; and, if chronic, would have eccentric hypertrophy of
the heart. Also, patients with infective endocarditis appear sick, and they will
have low to high-grade fevers, chills, or night sweats.

(Choice F) Asymmetric septal hypertrophy is seen in hypertrophic


cardiomyopathy. Hypertrophic cardiomyopathy is a disease of the young, and is
less likely to present with acute cardiac failure, rather than sudden cardiac death
which is commonly seen.
Educational Objective:
Dilated cardiomyopathy may be seen following viral myocarditis (postpartum),
particularly following Coxsackie B infection. Diagnosis is made by
echocardiogram, which typically shows dilated ventricles and diffuse hypokinesia
resulting in systolic dysfunction (low ejection fraction).

18% of people answered this question correctly

Test #20 Q.Id = 1857 Question 26 Of 26 Division : Cardiology


Item Review
A 60-year-old man is brought to the ER by his wife because he lost
consciousness in the bathroom at night. He says that he woke up, went to the
bathroom to urinate, and fainted there. He rapidly recovered his consciousness
without any indication of disorientation. He has never had such an episode
before. He admits ‘problems with urination,’ including difficulty with initiating
urination and frequent awakening to void at night. He does not take any
medication. His past medical history is insignificant. He smokes 2 packs of
cigarettes per day and does not consume alcohol. His blood pressure is 130/80
mm Hg while supine, and 132/80 while standing. His heart rate is 70/min. His
physical examination is within normal limits. The ECG is normal. What is the
most probable cause of the syncopal episode in this patient?

A. Arrhythmia
B. Postural hypotension
C. Situational syncope
D. Seizure
E. Transient ischemic attack (TIA)
Explanation:

The most probable diagnosis in this patient is situational syncope related to


micturition. The scenario described (middle age or older male with prostatic
hypertrophy, who lost his consciousness after awakening and voiding at night) is
typical for this type of syncope. The pathophysiologic mechanism underlying
the situational syncope includes autonomic dysregulation, which can be partially
explained by straining and rapid bladder emptying. Cardioinhibitory and/or
vasodepressor mechanisms may be involved.

Cardiovascular causes like arrhythmia (Choice A) are less likely because there
are no signs of a structural heart disease on the physical examination and
the ECG.

Postural hypotension (Choice B) is unlikely because there are no signs of


orthostatic BP changes, as well as no obvious cause for the problem, such
as medications or hypovolemia.
A patient with no previous history and a rapid recovery of consciousness argue
against seizure (Choice D) as a cause of the episode.

Transient ischemic attack (Choice E) is a rare cause of syncope, and is usually


accompanied by symptoms of focal neurologic dysfunction (ataxia, paresis,
diplopia).

Educational objective:
Situational syncope should be considered in the differential diagnosis of syncopal
episodes. The typical scenario would include a middle age or older male, who
loses his consciousness immediately after urination, or a man who loses his
consciousness during coughing fits.

26% of people answered this question correctly

Test #20 Q.Id = 593 Question 27 Of 26 Division : Cardiology


Item Review
A 60-year-old female presents with dyspnea and ankle edema. Her dyspnea was
mild about 6 months ago and used to occur with moderate exertion but it has
now progressed to dyspnea even at rest. She denies any chest pain, syncope, or
palpitations. She does not smoke or drink alcohol. She is not suffering from
diabetes mellitus, hypertension or hyperlipidemia. Her vitals are, PR: 80/min; BP:
130/70 mm of Hg; Temperature: 37.1C(98.9F); RR: 14/min. Her examination is
significant for jugular venous distension, bilateral ankle edema, and tender
hepatomegaly. Chest auscultation shows bibasilar crackles. Heart sounds are
distant and there is no murmur. Chest x-ray shows mild cardiomegaly and a
right-sided pleural effusion. EKG shows low voltage QRS complexes and
nonspecific ST-T wave changes. Echocardiography shows symmetrical
thickening of the left ventricular wall. A diagnosis of restrictive cardiomyopathy is
being considered. Which of the following abnormalities is most likely to be
present in cases of restrictive cardiomyopathy?

A. Severe systolic dysfunction


B. Severe diastolic dysfunction
C. Supernormal ejection fraction
D. Dynamic outflow obstruction
E. Increased left ventricular volume
Explanation:

Severe diastolic dysfunction is present in restrictive cardiomyopathy due to rigid


and stiff ventricle wall. Systolic function is preserved in this condition.

Severe systolic dysfunction and increased left ventricle size are features of
dilated cardiomyopathy.

Supernormal ejection fraction is present in hypertrophic cardiomyopathy as is


dynamic outflow obstruction. Diastolic dysfunction is present in HCM, due to the
stiff, hypertrophied ventricle wall.

Educational Objective:
Understand the presentation of restrictive cardiomyopathy and know its
pathophysiology and hemodynamics

56% of people answered this question correctly

Test #20 Q.Id = 587 Question 28 Of 26 Division : Cardiology


Item Review
A 27-year-old black male has a history of repeated attacks of
dyspnea upon exertion. He does not smoke, nor does he drink
alcohol. He denies chest pain or syncope. His vitals are stable and
he is afebrile. On examination, there is no edema or jugular venous
distension. His lungs are clear to auscultation. Auscultation of
precordium reveals a III/VI harsh diamond shaped systolic murmur
best heard at the left lower sternal border. EKG shows evidence of
left ventricular hypertrophy. Echocardiography is performed which
shows asymmetrical septal hypertrophy and systolic anterior motion
of the mitral valve. Which of the following is true regarding the
inheritance of this disease?

A. Autosomal dominant
B. Autosomal recessive
C. X-linked recessive
D. X-linked dominant
E. Multifactorial
Explanation:

This patient is most likely suffering from hypertrophic


cardiomyopathy that has an autosomal dominant mode of
inheritance. More than 100 mutations have been identified and
more than 40% involve cardiac beta-myosin heavy chain gene on
chromosome 14. It has been found that about one third of the first-
degree relatives of the patients of familial HCM have evidence of
HCM. There are many sporadic cases of HCM that occur due to
spontaneous mutations. It has been found that about one third of
the first-degree relatives of the patients of familial HCM have
evidence of HCM.

Educational Objective:
HCM follows autosomal dominant inheritance.

42% of people answered this question correctly

Test #20 Q.Id = 605 Question 29 Of 26 Division : Cardiology


Item Review
A 65-year-old white male was diagnosed with congestive heart failure one year
ago. His ejection fraction (EF) was estimated at 30-35% (Normal is 55-65%). His
heart failure is secondary to chronic alcohol use. His other medical problems
include type II DM that is being treated with insulin. His other medications include
lisinopril, aspirin, and simvastatin. He had been doing well on these medications,
until 5 days ago, when he started noticing palpitations, increasing dyspnea even
at rest, and swelling of his ankles. He is today in your office for these symptoms.
His PR: 122/min irregularly irregular; BP: 100/70mm Hg; Temperature:
37C(98.6F). EKG shows new onset atrial fibrillation. You increase his lasix dose.
Which of the following is the most appropriate next step in the management of
this patient's congestive heart failure?

A. Start digoxin
B. Start metoprolol
C. Start spironolactone
D. Start verapamil
E. Give IV dobutamine
Explanation:

The most appropriate next pharmacological measure for the treatment of this
patient's CHF is the addition of digoxin to his regimen. Digoxin slows the
ventricular response rate in atrial fibrillation by slowing down AV conduction and
thus increasing the diastolic filling. It also has a beneficial effect in patients of
systolic dysfunction due to its positive inotropic effect. Thus any patient who has
atrial fibrillation and heart failure will benefit from digoxin; it should be considered
over before beta-blockers or calcium channel blockers.

Another next best choice in the management of this patient' which is not given in
the choices would be to anticoagulate the patient, as this is one of the most
important interventions in reducing the morbidity and mortality associated with
atrial fibrillation

Verapamil, as well as metoprolol, will slow down the ventricular response rate by
its negative dromotropic effect but it may worsen systolic dysfunction by its
negative inotropic effect. Beta-blockers are indicated for heart failure patients
who are NYHA class II or III with systolic dysfunction. Beta-blockers are
preferred over the digoxin or calcium channel blockers in patients with coronary
artery disease. Calcium channel blockers are not the best drugs for patients with
heart failure due to their negative inotropic effect.

Furosemide, a loop diuretic, should be given in increased doses at this stage to


remove the excess of fluid responsible for the congestive heart failure symptoms
in this patient. However, this is not going to slow down the ventricular response
rate in the setting of AF that is the main cause of decompensation in this patient.
Adding spironolactone is not going to provide enough diuresis at this time but it
may be useful chronically in patients with severe CHF by inhibition of aldosterone
pathways. Indeed some studies have shown an improvement in mortality with the
use of spironolactone in heart failure patients.

IV dobutamine is used when a patient is in cardiogenic shock. This patient's


heart rate is already high (dopamine causes tachycardia) and his BP is not very
low. So, using dopamine is not indicated.

Educational Objective:
Know how to manage a case of CHF exacerbation due to atrial fibrillation with a
rapid ventricular response. Digoxin is the drug of first choice in this situation.

42% of people answered this question correctly

Test #20 Q.Id = 1107 Question 30 Of 26 Division : Cardiology


Item Review
A 24-year-old man rushes into the ER complaining of severe chest pain. He says
he was fine until 30 minutes prior, when he developed sudden onset of chest
pain. He denies any palpitations, shortness of breath, cough or fever. He denies
any previous episodes of chest pain. Past medical history is unremarkable
except for an appendectomy done one year back. He denies the use of any
prescription or over the counter medications. He denies any illicit drug use, but
admits to alcohol intake. His father died at the age of 64 due to “some heart
problem” and his mother died of ovarian cancer. The patient is agitated and is
sweating profusely. His vital signs are as follows: PR: 104/min, B.P: 140/90 mm
Hg, and R.R: 14/min. Physical examination is normal except for dilated pupils
and a small amount of blood at the external nares. An EKG done shows ST
elevation in leads V1-V4. The most likely explanation of his symptoms would be?

A. Myocardial infarction from atherosclerotic vascular disease


B. Acute pericarditis from Coxsackie virus
C. Pneumonitis from Streptococcus pneumoniae
D. Drug induced vasospasm
E. Aortic dissection
F. Drug induced pulmonary infarction
Explanation:

Patients with cocaine intoxication usually present with anxiety, aggressiveness,


agitation, and psychosis or delirium in the setting of a recent cocaine use. In
addition, they could develop elevated or low blood pressure, tachycardia or
bradycardia, profuse sweating, pupillary dilatation, nausea or vomiting, and
insomnia. Overdoses can be fatal as these patients can develop cardiac
arrhythmias, myocardial infarcts, seizures or stroke. Not very frequently, these
patients present with formication also known as “cocaine bugs”, where the
patient thinks that there are bugs crawling all over him. Nosebleeds can occur in
patients who snort cocaine. Although a more complete workup would be
necessary before making a diagnosis, this question asks for the MOST LIKELY
diagnosis. This patient denies any illicit drug use, but it is common to find
patients who deny drug use and have positive urine toxicology screens.

Although this patient does have ST elevation in the anterior leads, considering
his age, and other features like pupillary dilatation and bleeding from the nose,
cocaine use seems more likely. However, a complete cardiac work up is
mandatory to rule out other cardiac causes for his symptoms. A chest x-ray,
cardiac enzymes and urine toxicology screen would be necessary to make an
accurate diagnosis.

For the same reasons as above, cocaine intoxication would be the most likely
diagnosis. Acute pericarditis would present with diffuse ST elevations (ST
elevations in all the leads) on EKG.

In the absence of cough, fever and shortness of breath, Pneumonitis would be


unlikely in this patient. Besides, with his EKG findings, pneumonitis can clearly
be ruled out.

Aortic dissection usually does not cause pupillary dilatation; however, those
patients may have severe chest pain and hypertension,

Drug induced pulmonary infarction would not cause ST elevation in V1 to V4.

Educational Objective:
Think of cocaine intoxication in a young patient presenting with chest
pain/myocardial infarction or stroke. Features of cocaine intoxication are cocaine
bugs, agitation, decreased appetite, dilated pupils, elevated or decreased blood
pressure, tachycardia or bradycardia, and sweating.

42% of people answered this question correctly

Test #20 Q.Id = 1569 Question 31 Of 26 Division : Cardiology


Item Review
A 45-year-old recently migrated Mexican farmer comes to your office because of
dyspnea and fatigue, since last 2 months. His vital signs are, BP: 126/80 mm Hg,
PR: 80/min, RR: 16/min and Temperature: 37C(98F). On examination, he has
pedal edema, elevated jugular venous pressure with positive Kussmaul’s sign
and increased abdominal girth with free fluid. Auscultation reveals reduced
intensity of apex beat with an early heart sound following S2. Jugular venous
pressure tracing shows prominent ‘x' and ‘y' descent. Which of the following is
the most likely cause for this patient’s symptoms?

A. Cor pulmonale
B. Tuberculosis
C. Viral infection
D. Psittacosis
E. Pneumoconiosis
Explanation:

The diagnosis in this patient is constrictive pericarditis. Constrictive pericarditis


results from obliteration of pericardial space and fibrosis of pericardium following
an acute fibrinous or serofibrinous pericarditis or chronic pericardial effusion.
Tuberculosis is a very common cause of constrictive pericarditis, especially in
developing countries. This patient is a recently migrated Mexican farmer, which
puts TB on the top of the list of differentials for his constrictive pericarditis
(Option E). Some other causes of constrictive pericarditis are idiopathic (42% in
USA), post radiotherapy (31% in USA), post surgical (11%), connective tissue
disorders, neoplasm, uremia, sarcoidosis, etc.

Constrictive pericarditis will lead to inability of ventricle to fill properly during


diastole and would further cause the signs and symptoms of decreased cardiac
output (fatigue, muscle wasting etc) and /or signs and symptoms of venous
overload like elevated JVP, dyspnea, ascites, positive Kussmaul's sign, pedal
edema, tender hepatomegaly etc. Sharp 'x' and 'y' descent on central venous
tracing is characteristic of constrictive pericarditis as is the presence of
pericardial knock (early heart sound heard after S2).

Option D and E: The clinical presentation of this patient is highly suggestive of


constrictive pericarditis and psittacosis and pneumoconiosis are not an
etiological cause of this condition.

Option A: This patient has no finding suggestive of chronic lung disease, which
would lead to cor pulmonale. Also cor pulmonale would not cause constrictive
pericarditis.

Educational objective:
Tuberculosis is the most common cause of constrictive pericarditis, in immigrant
population. It should be considered in patients with unexplained elevation of JVP
and history of predisposing condition.

16% of people answered this question correctly


Test #20 Q.Id = 1701 Question 32 Of 26 Division : Cardiology
Item Review
45-year-old male presents to your clinic with complaints of pleuritic chest pain.
He says he is a heavy smoker and has been having intermittent cough for the
past 2 weeks. His blood work is normal and his chest-x ray is clear. EKG reveals
a normal sinus rhythm with a heart rate of 68. He does have a PR interval > 0.2
seconds. The next step in the management of his arrhythmia is:

A. Observation
B. Atenolol
C. Pacemaker
D. Electrophysiology study
E. Cardiac catheterization
F. Atropine
Explanation:

First-degree heart block is characterized by a prolonged constant PR interval


(>0.2s). The heart block may be entirely normal or may be secondary to
increased vagal tone or use of digitalis. The AV conduction must be normal and
intact. When first-degree heart block is identified, no treatment is required. The
majority of patients are asymptomatic and remain so. Almost all cases of first-
degree heart block are discovered as an incidental finding while working up the
patient for a different disorder.

Option A: First-degree heart block requires no treatment. The heart rate is not
affected and neither bradycardia nor any form of AV block is seen. Atropine,
which is an anticholinergic agent, is never used to treat first-degree heart block.
Atropine increases heart rate.

Option B: Beta-blockers have no role in the treatment of heart block of any form.
Beta-blockers can precipitate and worsen heart block. First-degree heart block
does not affect the heart rate or the force of contraction. Beta-blockers are
generally used as anti-hypertensives, anti anginals and anti arrhythmics.

Option C: Pacemakers are never inserted for first-degree heart block.


Pacemakers are more suitable for bradycardia or trifascicular blocks.

Option D: When an individual is diagnosed with a first-degree heart block, no


further studies are required. The disorder is benign and does not progress to
heart block of any form. If the patient has been on digoxin, digoxin levels may be
ordered or the drug with held for a few days. Electrophysiological studies are
generally done when the site and focus of arrhythmias is required for ablation or
resection. EP studies are almost never done for first-degree heart block, as there
is no pathology ever identified.
Option E: Catheterization is an invasive procedure and certainly not indicated for
this benign arhythmia.

Option F: Atropine is used for symptomatic bradycardia resulting from type II


and type III heart block, not for type I block.

Educational objective:
First-degree heart block is a completely benign arrhythmia and requires no
treatment.

41% of people answered this question correctly

Test #20 Q.Id = 915 Question 33 Of 26 Division : Cardiology


Item Review
A 45-year-old white male is brought to the emergency room with sudden onset of
substernal chest pain. Pain is 6-8/10 in severity, it is alleviated by leaning forward
and worsened by coughing and deep breathing; he says the pain goes to his left
arm. He had an upper respiratory tract infection 1 week ago. He is a non-smoker
and does not drink alcohol. He denies any cough or dyspnea. His vitals are
stable except for a mild fever. Chest examination is significant for a pericardial
friction rub. Chest-X ray is normal and EKG shows diffuse ST segment elevation
with ‘J’ point elevation, no ‘Q’ waves, and no reciprocal changes. Which of the
followings is the most likely cause of his symptoms?

A. Coxsackie virus
B. Mycobacterium tuberculosis
C. Borrelia burgdorferi
D. Uremia
E. Streptococcal pneumoniae
F. Coronary artery disease
Explanation:

Severe constant pain that localizes over the anterior chest, radiates to the arm,
shoulder, back, epigastrium or neck, is intensified with inspiration, and relieved
by sitting up and leaning forward represents a classic presentation of acute
pericarditis. The pain is often difficult to differentiate from an ischemic pain
because often it is very severe, short, and sometimes has a constricting quality
that radiates either to one or both arms. However, pain which is relieved by
sitting up and leaning forward and is intensified by inspiration or lying supine is
characteristic of acute pericarditis. Acute pericarditis has a very broad differential
diagnosis. But it is most commonly caused by viruses and Coxsackie virus is a
frequent offender. A history of a recent viral illness is common. Other viruses
implicated include echovirus, adenovirus, EBV, and HIV. The classic findings of
pericarditis on EKG are:
1.Diffuse ST segment elevation with upward concavity at ‘J’ point.
2.No new ‘Q’ waves
3.'PR’ segment elevation in aVR with ‘PR’ depression in other leads.

Mycobacterium tuberculosis is an important cause of pericarditis but it is very


rare in the absence of other risk factors thus highly unlikely in this patient.

Uremia can cause acute pericarditis and this may represent an indication for
dialysis. The above patient does not have any clue about renal insufficiency.

The above patient does not have pneumonia.

B Burgdorferi, the agent of Lyme disease, may cause an acute myopericarditis


but is not likely to cause isolated acute pericarditis. Cardiac involvement in Lyme
disease occurs in 8% of patients with Lyme disease and usually the most
common type of it is fluctuating degrees of atrioventricular block.

Educational Objective:
Recognize the clinical presentation of acute pericarditis. Coxsackie-B virus is a
common cause of acute pericarditis.

57% of people answered this question correctly

Test #20 Q.Id = 31 Question 34 Of 26 Division : Cardiology


Item Review
A 65-year-old white female, with chronic congestive heart failure, was admitted
for community-acquired pneumonia. She is receiving aspirin, digoxin,
furosemide, levofloxacin, and oral simvastatin. She is feeling better and her
pneumonia is resolving. While planning for discharge she developed recurrent
sustained ventricular tachycardia with stable vital signs during the nighttime. The
patient has recieved amiodarone and now is stable in sinus rhythm. A recent
echocardiogram showed an ejection fraction (EF) of 35%. What is the most
important next step in the management at this stage?

A. Add beta blocker


B. Add spiranolactone
C. Measure serum electrolytes
D. Stop simvastatin
E. Discharge the patient
Explanation:

When someone is having recurrent VT, first thing to do after stabilizing the
patient is to search for underlying cause. This patient, most likely, has an
electrolyte imbalance due to diuretics. Furosemide commonly causes
hypokalemia, which may lead to digoxin toxicity. Therefore, ordering serum
electrolytes and serum digoxin level is the most reasonable approach.

Adding a beta-blocker is a good thing for chronic heart failure but not the next
step in this case.

Simvastatin has nothing to do with ventricular tachycardia.

Spironolactone is a potassium sparing diuretic that is not indicated at this stage.

Educational Objective:
Look for electrolyte abnormalities and correct them in patients with arrhythmias.

31% of people answered this question correctly

Test #20 Q.Id = 644 Question 35 Of 26 Division : Cardiology


Item Review
A 43-year-old white male is found to have atrial premature complexes (APCs) on
a routine EKG. The patient denies any chest pain, shortness of breath, or
syncopal episodes. He has been smoking cigarettes 1-2 packs/day for the past
20 years. He has been an alcoholic for 20 years and still drinks 1-2 beers/day.
His family history is significant for his mother having had an MI at the age of 65
and his father having had a stroke at the age of 72. He has no personal history of
hypertension or diabetes. A complete physical examination is unremarkable. His
vitals are BP: 130/70 mm Hg; PR: 80/min and afebrile. Which of the following
interventions is the most appropriate regarding the management of this patient?

A. Reassurance, and send the patient home


B. Place the patient on 24-hour Holter monitor
C. Do an echocardiogram
D. Start a beta blocker
E. Advise him to stop alcohol and tobacco
Explanation:

An atrial premature beat results from the premature activation of the atria that
originates from a site other than the SA node. EKG shows an early P wave. Atrial
premature beats may be present with underlying heart disease or may even be
present in the absence of any underlying heart disease. They are usually
asymptomatic or can cause palpitations. They may be single or may be seen in
the pattern of bigeminy. Occasionally they may result in supraventricular or less
commonly ventricular arrhythmias.

Treatment is required only when the patient with atrial premature contraction has
disturbing palpitations or supraventricular tachycardia. Precipitating factors like
tobacco, alcohol, or adrenergic agonists need to be identified and corrected
before opting for pharmacological therapy with beta-blockers or calcium channel
blockers. The above patient has precipitating factors that need to be corrected.
He is asymptomatic and does not have supraventricular tachycardia; therefore
there is no need for pharmacological treatment.

Reassurance alone is not appropriate as the identified risk factors for the
development of atrial premature contractions need to be removed.

This patient has no valvular, coronary artery disease or heart failure and
echocardiography is therefore not required.

Holter monitoring is used both for diagnostic and prognostic purposes. In the
above case, diagnosis is already established and his arrhythmia so far is benign.
Therefore, there is no need for Holter monitoring.

Educational Objective:
Tobacco and alcohol are reversible risk factors for the development of atrial
premature beats.

43% of people answered this question correctly

Test #20 Q.Id = 980 Question 36 Of 26 Division : Cardiology


Item Review
A 34-year-old Caucasian male, who has a 10 year history of asymptomatic Wolff-
Parkinson-White syndrome, presented with 4 hours history of palpitations. He
denies any chest pain, shortness of breath, fevers, or chills. On examination, he
has an irregularly irregular pulse. EKG shows atrial fibrillation with a rate of
110/min associated with Wolff-Parkinson-White syndrome. His BP: 120/80
mmHg; RR: 18/min; Temperature: 37C(98.6F). Which of the following
interventions is the next most appropriate in the management of this patient?

A. IV digoxin.
B. IV verapamil.
C. IV lidocaine.
D. IV procainamide.
E. Cardioversion.
Explanation:

The drugs that slow the AV conduction (digoxin and verapamil) are
contraindicated in patients with atrial fibrillation and Wolff-Parkinson-White
syndrome because they can increase the conduction of impulses through the
accessory pathway, thus leading to malignant arrhythmias and hypotension.

Lidocaine, as well in some cases, may worsen the situation by increasing the
conduction through the AV node. That is why lidocaine is usually not a preferable
drug in this situation.

Cardioversion is usually indicated in patients who are hemodynamically unstable


(very rapid ventricular rates with hypotension). In those circumstances the likely
chances of conversion to normal sinus rhythm with drug therapy is very low.
However, this patient is relatively stable with a normal blood pressure so these
patients are more likely to respond with IV procainamide and it is the drug of
choice in these circumstances. You can also use IV disopyramide.

Educational Objective:
Procainamide or disopyramide are the drugs of choice for Atrial fibrillation in the
context of WPW syndrome. Remember digoxin and calcium channel blockers
should not be used.

17% of people answered this question correctly

Test #20 Q.Id = 2037 Question 37 Of 26 Division : Cardiology


Item Review
A 20-year-old female presents to your office for a routine check-up. She has no
complaints and her past medical history is insignificant. She is not taking any
medications and denies drug abuse. Her blood pressure is 125/65 mmHg and
heart rate is 80/min. Cardiac auscultation reveals early diastolic murmur at the
left sternal border. What is the next best step in the management of this patient?

A. Electrocardiogram
B. Chest x-ray
C. Coronary angiography
D. Echocardiography
E. No further work-up
Explanation:

Diastolic and continuous murmurs revealed on cardiac auscultation should


always be investigated, because the probability that an organic cause is present
is high. Midsystolic soft murmurs (grade I-II/IV) in an asymptomatic young patient
are usually benign and need no further work-up (Choice E). Transthoracic
Doppler echocardiography is a non-invasive and very efficient tool to confirm the
presence of flow abnormalities, find the cause, and assess the extent of the
problem.

Catheterization and angiography (Choice C) can also reveal the structural defect
and assess the severity of the process, but it is invasive; therefore,
echocardiography is employed first.

Chest x-ray (Choice B) and electrocardiogram (Choice A) are too non-specific


and have limited use in such a case.

Educational Objective:
Diastolic and continuous murmurs as well as loud systolic murmurs revealed on
cardiac auscultation should always be investigated using transthoracic Doppler
echocardiography. Midsystolic soft murmurs (grade I-II/IV) in an asymptomatic
young patient are usually benign and need no further work-up.

33% of people answered this question correctly

Test #20 Q.Id = 2029 Question 38 Of 26 Division : Cardiology


Item Review
A 47-year-old male was brought to the emergency room with chest pain of acute
onset. The pain was associated with nausea, vomiting, and diaphoresis. He has
a history of diabetes, hypertension, and hyperlipidemia. ECG reveals ST
segment elevation in the anterolateral leads and ventricular premature beats
(VPBs). The patient dies within the first hour after the arrival to emergency room.
What is the most likely pathophysiologic mechanism responsible for this patient’s
death?

A. Electro-mechanic dissociation
B. Reentry
C. Full conduction block
D. Increased automaticity
E. Asystole
Explanation:

The most common cause of death in patients with acute myocardial infarction
(MI) is complex ventricular arrhythmia. Acute ischemia creates heterogeneity of
conduction in the myocardium. Areas of partial block of conduction are frequently
formed that predispose the patient to reentrant arrhythmia. Ventricular fibrillation
is a typical example of reentrant arrhythmia. Decline in mortality of patients
hospitalized with acute coronary syndromes is largely attributable to the effective
detection and treatment of reentrant ventricular arrhythmias.

Full conduction block (Choice C) may occur in patients with acute myocardial
infarction, especially inferior wall MI, but it is a less common cause of death.

Asystole (Choice E) usually occurs in patients with severe complications of MI


(e.g., heart failure).

Electromechanical dissociation (Choice A) is typical for pulmonary


thromboembolism and pericardial tamponade.

Increased automaticity (Choice D) is a frequent cause of arrhythmia in patients


with glycoside intoxication.

Educational Objective:
Reentrant ventricular arrhythmia (ventricular fibrillation) is the most common
cause of death in patients with acute myocardial infarction.

4% of people answered this question correctly

Test #20 Q.Id = 24 Question 39 Of 26 Division : Cardiology


Item Review
A 63-year-old Caucasian male is admitted for sudden onset of severe chest pain.
His ECG revealed ST elevation in leads V2-V6. He was given thrombolytic
therapy, heparin, aspirin, metoprolol, morphine, and nitrates. His coronary
angiogram, after thrombolytic therapy, revealed a 50% obstruction in the left
anterior descending artery. On the 3rd day of his hospitalization the patient
suddenly developed severe shortness of breath at rest and quickly became
hypotensive. Examination reveals a soft S1, an apical pansystolic murmur (PSM)
radiating to the axilla, and bibasilar crackles. His vitals are BP: 90/60 mm of Hg;
HR: 102/min; RR: 30/min; Temperature: 37.8C (100F). An echocardiogram
performed on the 2nd hospital day revealed an anterior akinetic segment. What
is the most likely explanation for this patient's deterioration?

A. Pericardial tamponade
B. Pulmonary embolism
C. Rupture of ventricular septum
D. Papillary muscle dysfunction
E. Acute aortic dissection
Explanation:

This patient has classic features of new onset mitral regurgitation. Sudden onset
of shortness of breath (SOB), bibasilar rales, and an 'apical' murmur radiating to
the axilla are quite characteristic of new onset mitral regurgitation. Papillary
muscle dysfunction, or rupture, is the most common cause of MR in this setting.

Ventricular septal rupture has similar features but the murmur is heard at the left
sternal border and would not radiate to the axilla.

Pericardial tamponade is a very important differential diagnosis in this patient but


usually patients will not have any murmurs.

Pulmonary embolism (PE) and aortic dissection are completely different from this
presentation. Pulmonary edema is not seen in PE.

Educational Objective:
Recognize the early complications of an acute anterior wall MI.

50% of people answered this question correctly

Test #20 Q.Id = 619 Question 40 Of 26 Division : Cardiology


Item Review
A 56-year-old male, who is being treated with sildenafil for his sexual
dysfunction, develops intermittent substernal chest pain that radiates to his jaw
and left upper arm. He describes the pain as a sense of heaviness, is 7/10 in
severity, is brought on by exertion, and is relieved by rest. His past medical
history is insignificant other than his sexual dysfunction. He is a non-smoker and
non-alcoholic. EKG performed while having chest pain in the emergency
department shows T-wave inversion in leads II, III and aVF. Cardiac enzymes
are normal. Which of the following is true regarding the treatment of his ischemic
symptoms with nitrates?

A. Nitrates should not be given to him within 24 hours of the last dose of sildenafil
B. Nitrates should not be given to him within 12 hours of the last dose of sildenafil
C. The dose of nitrates should be reduced
D. Nitrates can be given safely without any change in the usual nitrate dose
E. The dose of nitrates should be increased
Explanation:

Nitrates are contraindicated when a patient is continuously or intermittently taking


sildenafil (Viagra). In such settings nitrates may cause syncope, MI, or sudden
death when a patient has an acute coronary syndrome. It is recommended not to
use nitrates within 24 hours of the last dose of sildenafil. The reason for the
dangerous interaction between nitrates and sildenafil is that both induce nitric
oxide mediated vasodilatation.

Reduction of the nitrate dose would not protect against the dangerous interaction
with sildenafil and is not the right choice here.

Within 12 hours of the last dose of sildenafil, considerable quantities of sildenafil


could still be present in the circulation and thus it would dangerously interact with
the nitrates.

An increase in the nitrates dose, or giving nitrates at the same dose in patients
who are taking sildenafil, is likely to produce substantial vasodilatation resulting
in syncope, MI, or even sudden death.

Educational Objective:
Nitrates are contraindicated when a patient is continuously or intermittently taking
sildenafil (Viagra).
37% of people answered this question correctly

Test #20 Q.Id = 26 Question 41 Of 26 Division : Cardiology


Item Review
A 57-year-old female with known bronchial asthma presents to your office for
evaluation of her HTN for the 3rd time. Her last 2 visits revealed BP: 170/90 and
174/92 after 10 min of rest. Her vitals for this visit are: BP: 170/90 mm Hg, HR:
78/min, RR: 18/min, Temperature: 37.8C (100F). She denies headaches and any
other complaints. She has been on beclomethasone and albuterol inhalation for
her chronic persistent asthma. She denies smoking or alcohol intake. She has no
allergies. Her family history is significant for myocardial infarction in both her
mother and father. She tried regular exercise and low salt diet without much
improvement. Based on this information, what step would you take next?

A. Ask her to stop asthma medications


B. Prescribe propranolol
C. Prescribe enalapril
D. Prescribe hydrochlorothiazide
E. Do echocardiogram
Explanation:

The first line medication for HTN in the general population is either a thiazide
diuretic or beta-blocker. Non-selective beta-blockers are contraindicated in
asthmatics especially if they are steroid dependent or on continuous albuterol.
Stopping the asthma medications is not a good choice here. Use of propranolol
is also contraindicated in this patient.

ACE inhibitors, like enalapril, are the 1st line drugs for diabetics with
hypertension. These are usually 2nd line drugs for the general population.

Usually an echocardiogram is not required in an asymptomatic patient, although


she may have concentric hypertrophy.

Therefore the best choice in this patient is hydrochlorothiazide.

Educational Objective:
Choose the appropriate initial antihypertensive therapy in a patient with asthma.
HCTZ is the initial drug of choice in patients with chronic persistent asthma.

38% of people answered this question correctly

Test #20 Q.Id = 2062 Question 42 Of 26 Division : Cardiology


Item Review
A 40-year-old female presents to the emergency room with palpitations and
lightheadedness of acute onset. Also, she has experienced insomnia, fatigability,
and weight loss lately. She does not smoke or consume alcohol. She is not
taking any medications. Her blood pressure is 110/80 mmHg and heart rate is
120/min, irregular. Physical examination reveals lid lag and fine tremor of the
outstretched hands. ECG shows atrial fibrillation with rapid ventricular response.
What is the next best step in the management of this patient?

A. Digoxin
B. Propranolol
C. Lidocaine
D. Quinidine
E. Immediate cardioversion
Explanation:

This patient presents with signs and symptoms suggestive of Grave’s disease -
insomnia, fatigability, weight loss, lid lag, and tremor. Atrial fibrillation is a
common complication of hyperthyroidism. As in other cardiovascular
complications, atrial fibrillation in patients with hyperthyroidism is believed to be
caused by increased sensitivity of beta-adrenoreceptors to sympathetic stimuli.
The best initial choice for these stable patients is a beta-blocker like propranolol.
It not only helps to control the rhythm in tachysystolic atrial fibrillation, but also
diminishes other symptoms of hyperthyroidism.

Digoxin (Choice A) is not a good choice, because rapid ventricular response in


hyperthyroidism-related atrial fibrillation is resistant to cardiac glycosides.

Quinidine (Choice D), when used without pretreatment with rhythm-controlling


drugs, may even worsen tachysystoly. It is not commonly used today for
chemical cardioversion.

Electrical cardioversion (Choice E) is effective in restoring the sinus rhythm; but,


if the underlying condition (hyperthyroidism) is not addressed, atrial fibrillation will
most probably recur. Emergent cardioversion is indicated if the arrhythmia
precipitates dangerous complications (like hypotension, angina, or heart failure),
or if the patient is hemodynamically unstable.

Lidocaine (Choice C) is useless in this situation, because it is not effective in


atrial arrhythmias.

Educational Objective:
In patients with hyperthyroidism-related tachysystolic atrial fibrillation, a beta-
blocker is the drug of choice.

15% of people answered this question correctly


Test #20 Q.Id = 1839 Question 43 Of 26 Division : Cardiology
Item Review
A 63-year-old female presents to your office for a routine check-up. She has no
present complaints. Her past medical history includes DM, type 2, and
hypertension. Her current medications include glyburide and atenolol. She does
not smoke. She drinks 2-3 glasses of wine 1-2 times a week. Three consecutive
BP measurements were in the range of 138-142/87-90 mmHg. Physical
examination is within normal limits. Her recent fasting glucose level was 250
mg/dL. ECG recorded 1 month ago showed left ventricular hypertrophy. Which
statement about the BP control in this patient is the most accurate?

A. BP is within acceptable range


B. BP is within optimal range
C. It is better to keep systolic pressure < 130 mmHg to slow end-organ damage
D. Diastolic BP is within acceptable range, but systolic is not
E. Systolic BP is within acceptable range, but diastolic is not
Explanation:

Traditionally a goal, blood pressure is considered to be below 140/90 mmHg. But


it is recently recognized that BP needs more tight control in diabetics and
patients with chronic renal failure. These two groups of patients are especially
sensitive to high BP, that’s why the values of systolic BP for these patients
should be kept lower than 130 mmHg and diastolic BP lower than 80 mmHg to
prevent end organ damage.

Blood pressure could be considered in acceptable range (Choice A) if the


patient was not diabetic. According to JNC-7 report (JNC stands for Joint
National Committee on Prevention, Detection, Evaluation, and Treatment of High
Blood Pressure) BP is considered optimal when it is below 120/80 mmHg
(Choice B). In this patient both systolic (Choice E) and diastolic (Choice D) BP
are above the goal values.

Educational objective:
It is better to keep systolic pressure < 130 mmHg to slow end-organ damage in
patients with diabetes and chronic renal failure.

24% of people answered this question correctly

Test #20 Q.Id = 1913 Question 44 Of 26 Division : Cardiology


Item Review
A 40-year-old African American male presents to the Emergency Room with a
two-month history of occasional severe headache and blurring of vision. His past
medical history is significant for hypertension controlled with hydrochlorothiazide
for two years. His family history is significant for hypertension and diabetes. He
smokes two packs a day and occasionally consumes alcohol. His blood pressure
is 200/140 mmHg and heart rate is 75/min. Which of the following is most
consistent with a diagnosis of malignant hypertension in this patient?

A. Left ventricular hypertrophy on ECG


B. Elevated serum creatinine level
C. Papilledema
D. Oliguria
E. Blood pressure >/= 200/140 mmHg
Explanation:

Ophthalmoscopy is an important component of the evaluation of patients with


hypertension. Presence of papilledema in this patient is consistent with a
diagnosis of malignant hypertension. Although the blood pressure of the patients
with malignant hypertension is usually >/= 200/140 mmHg (Choice E),
papilledema should be present in order to make a diagnosis of malignant
hypertension. Renal failure with elevated creatinine level (Choice B) and oliguria
(Choice D) can develop rapidly in this case without treatment, but its presence is
not required to diagnose malignant hypertension. Left ventricular hypertrophy on
ECG (Choice A) is not a diagnostic criterion for malignant hypertension.

Educational Objective:
Always suspect malignant hypertension in patients with very high blood pressure
(>/= 200/140 mmHg). Presence of papilledema on ophthalmoscopy confirms the
diagnosis. The pathologic change responsible for end-organ damage in
malignant hypertension is fibrinoid necrosis of small arterioles.

17% of people answered this question correctly

Test #20 Q.Id = 608 Question 45 Of 26 Division : Cardiology


Item Review
56-year-old white male presents to the emergency room with central chest pain
of sudden onset that woke him up from sleep. Pain is 9/10 in severity, feels like a
pressure sensation, radiates to left upper arm and lower jaw, and is not relieved
by sublingual nitroglycerin. He has 40 pack year history of smoking and he is an
occasional alcohol drinker. His HTN was diagnosed 2 years ago and he is not
taking any medication for it. He is anxious and sweating profusely. His lungs are
clear to auscultation. Acute MI is a strong consideration on your differential
diagnosis. Which of the following would you expect to be the earliest EKG finding
in a patient with an acute MI?

A. Peaked 'T' wave


B. 'ST'segment elevation
C. 'T' wave inversion
D. Appearance of 'Q' wave
E. 'ST' depression with 'T' inversion
Explanation:

The earliest EKG finding in acute MI is peaked (hyperacute) T waves, followed


by ST segment elevation, followed by the inversion of T waves, followed by the
appearance of Q waves.

Q waves do not occur in 30-50% cases of acute infarcts (Non Q wave MI).

The earliest changes of hyperacute ‘T’ waves are frequently not seen in clinical
practice because by the time the patient present they already have ‘ST’
elevation.

Educational Objective:
Know the progression and timeframe of the ECG findings in the setting of an
acute MI. The earliest EKG finding in acute MI is peaked (hyperacute) T waves.

17% of people answered this question correctly

Test #20 Q.Id = 601 Question 46 Of 26 Division : Cardiology


Item Review

A 60-year-old woman presents to the emergency room for the acute onset of
breathlessness. She has had similar episodes on three prior occasions over the
last few months. Her past medical history is significant for rheumatic fever as a
child. Her vitals are, PR: 115/min and irregularly irregular; BP: 130/70 mm of Hg;
Temperature: 37.1C(98.7F); RR: 14/min. Her apex beat is tapping in quality and
is in left fifth intercostal space just inside the mid-clavicular line. Cardiac
auscultation shows a loud first heart sound and a mid-diastolic rumble best heard
at the apex. Auscultation of her lungs reveals crepitations in both lung fields.
Based on these findings, what is the most likely diagnosis of this patient?

A. Mitral stenosis
B. Mitral regurgitation
C. Tricuspid stenosis
D. Aortic stenosis
E. Aortic insufficiency
Explanation:

This patient most likely suffers from mitral stenosis, which is the most frequent
valvular disease caused by rheumatic heart disease. The presence of a tapping
apex beat, a loud first heart sound, and a mid-diastolic rumble at the apex
suggest the diagnosis of mitral stenosis in this patient. Atrial fibrillation is quite
common in patients with mitral stenosis.

Mitral regurgitation may also occur in rheumatic heart disease but it is


characterized by a pansystolic murmur best heard at the apex that radiates to the
axilla.

Aortic stenosis, aortic insufficiency, and tricuspid stenosis occur less frequently in
the setting of rheumatic heart disease and the auscultatory findings
in this patient are not compatible with any of these disorders.

The murmur of aortic regurgitation occurs in early diastole, is decrescendo, with


a high-pitched, blowing quality and is best heard in the left third intercostal
space. The murmur intensifies by leaning forward and holding the breath in
expiration.

The murmur of aortic stenosis occurs in systole (systolic ejection murmur), and
radiates to the neck.

The murmur of tricuspid stenosis occurs in mid to late diastole (presystolic), it


increases in inspiration, and is better heard at the left lower sternal border or in
the epigastrium. When atrial fibrillation is present, a mid-diastolic rumble can be
heard.

Educational Objective:
Recognize the clinical signs of mitral stenosis, and know how to differentiate it
clinically from other valvulopathies.

64% of people answered this question correctly

Vous aimerez peut-être aussi